Survey Finds High Fees Common in Medical Care

A patient in Illinois was charged $12,712 for cataract surgery. Medicare pays $675 for the same procedure. In California, a patient was charged $20,120 for a knee operation that Medicare pays $584 for. And a New Jersey patient was charged $72,000 for a spinal fusion procedure that Medicare covers for $1,629.

The charges came out of a survey sponsored by America’s Health Insurance Plans in which insurers were asked for some of the highest bills submitted to them in 2008.

The group, which represents 1,300 health insurance companies, said it had no data on the frequency of such high fees, saying that to its knowledge no one had studied that. But it said it did the survey in part to defend against efforts by the Obama administration to portray certain industry practices as a major part of the nation’s health care problems.

The health insurers, saying they felt unfairly vilified, gave the report to The New York Times before posting it online on Tuesday, explaining that they wanted to show that doctors’ fees are part of the health care problem.

Read it all.

print

Posted in * Culture-Watch, Health & Medicine

98 comments on “Survey Finds High Fees Common in Medical Care

  1. St. Jimbob of the Apokalypse says:

    I wonder if the high prices are compensating for the wage-slave rates that Medicare pays doctors and nurses for various procedures. Even our BCBS coverage only paid our Certified Nurse/midwife only $120 for assisting during a C-Section last year. She even helped my wife start nursing the baby 30 minutes after delivery. She’s been worth way more than that for the 4 children she’s helped us deliver over the years.

    At least we could choose her, the practice she’s associated with, and the Catholic hospital where all our children have been born. No telling what we’ll end up with when the government takes over.

  2. Chris Molter says:

    [blockquote]doctors’ fees are part of the health care problem.[/blockquote]
    mmhmm. and doctors will say that insurers’ premiums for malpractice insurance inflate their fees. Then those insurers will say that the sad state of malpractice law leads to them having to charge higher premiums due to settlement costs, frivolous suits, etc.

    The problems are much more complex than “doctor’s fees”

  3. Clueless says:

    The problem is that Medicare pays less than the cost of most surgeries, and Medicaid way less. Doctors bill private insurers more in order to pay for Medicare and Medicaid. If private insurers pay what medicare pays, then hospitals will simply close.

    The instruments that are placed in a spinal fusion (the hardware that is left in the spine to stabilize it) cost about 2,000. This does not count the physician’s time, the OR nurses time, the anesthesiologists time, the cost of maintaining a sterile operating room, a recovery room, an ICU, IV fluids, medications, anesthetics and the rest. If there was one bill fits all (in a low rent district, not New Jersey) and if physicians, nurses, hospital administrators and all other hospital personnel were paid minimum wage one could probably do a spinal fusion for about 15,000. That is about what it would cost if you did the medical tourism thing in India.

    If one subsidizes medicare and medicaid, one ends up with high prices for Aetna and Blue Cross. This is what has happened.

  4. Lee Parker says:

    There are definitely issues with fees and it is a huge part of the problem. You can have substantially different fee structures depending on whether you have insurance or not. Yes the problems are complex but we can substantially improve the system. It will not happen without government intervention.

  5. Archer_of_the_Forest says:

    I am convinced that one way to get a handle on the whole healthcare thing is to deal with exorbitant price gouging on the parts of healthcare providers. My wife recently had a baby, and the bills for that have been trickling in the last month or two. Its amazing how much they overcharge for stuff. I mean, $100 for a swaddling blanket? $25 for cafeteria meals for the non-hospitalized spouse?

    And there is no way to appeal such charges that I can see, and its outright price gouging. Luckily my insurance picked up the tab on most of that, but it was the principle of the thing. I think if we could create some sort of oversight on that end, the price spiral could be controlled more on the other end.

  6. Lee Parker says:

    This is the tip of the iceberg Archer and you are so right. When you say insurance “picked it up” you are understating. Trust me that cost is being passed on to all of us.

  7. Lee Parker says:

    BTW, Kendall I picked up a name from one of your articles last week and contacted a “Health Care” Economist here locally. There is lots of great data out there.

  8. Katherine says:

    A fair amount of the problem is because of government intervention (Medicare and Medicaid). More government intervention won’t fix it; it will make it worse. Note that the insurance companies have an interest in looking at fees charged and questioning the excessive ones. Medicare claims are paid at flat fees at what is often below the cost to the provider, much less any profit (doctors and nurses need to eat too).

  9. Sarah1 says:

    RE: “A fair amount of the problem is because of government intervention (Medicare and Medicaid). More government intervention won’t fix it; it will make it worse.”

    Yep, you got that right. In fact, the very reason why we — privately insured folks — pay ghastly prices is because the government FORCES hospitals to care for those without any insurance and with merely Medicare and Medicaid which do not pay the barest costs of delivery of the services.

    So the Lee Parkers, the Katherine’s and the Sarah’s are paying for Medicare and Medicaid NOT merely through our ever soaring taxes, but also through the charges given to us by hospitals and elsewhere. One of the primary causes of medical inflation is precisely medicare and medicaid. That, along with the FDA issues, tort issues driving physicians and practices out of state, and the fact that insurance policies are attached within *corporate* entities [work] rather than *individuals* adds up to a huge stew of annual Weimar-Republic inflation of medical costs.

  10. Lee Parker says:

    Unfortunately, government intervention will be the only way to restructure the system. The private system is close to destroying and greedily gobbling up my family’s future. And yes physicians are a big part of the problem. It is somewhat rare to find one who is not arrogant and operates without a sense of entitlement. I believe the answer will be two or three tiered but something must be done before it is too late.

  11. Chris says:

    Lee, you need to meet more doctors if you think it’s “somewhat rare to find one who is not arrogant and operates without a sense of entitlement.” And I am not a doctor, nor even in the health care industry….

  12. Sarah1 says:

    RE: “The private system is close to destroying and greedily gobbling up my family’s future.”

    No, as has been pointed out to you and others numerous times — the private system is merely attempting to compensate for the *State system* which the private system is *forced* to support — and all that some keep saying is “let’s throw more State at the problem.”

    That’s fine — but that means that things will merely get worse, not better. There will be more State, less care, and far more expense.

    Ah well . . . c’est la vie. If people can’t recognize what *the State* has done to healthcare over the past 30 years, than they will be doomed to repeat it — and “just add more.”

  13. Jeffersonian says:

    Funny, I never get gouged like this for anything else I buy: cars, TVs, milk, carpeting, cell phones, barbecue sauce, etc. I don’t even get raked over the coals on other insured things like collision repair. Why is it that pricing is so distorted and wildly variable in medical care?

  14. Katherine says:

    Lee Parker, I think that characterization of doctors is grossly unfair, and yes, we’ve had a few experiences with missed diagnoses or inadequate treatment. Physicians are human.

    You want government intervention to restructure the system, but if you think things are bad now, wait until the government is totally in charge.

  15. Pb says:

    Major tort reform is the item of business. Obama will not touch it. The doctors have this wrong too. It is not frivolous litigation that is causing the problem. It is the legitimate claims of folks who have been seriously iinjured by the system. We need some sort of workers comp type system where the costs are built in and recovery somewhat liimited. Defensive medicine runs up the bill. A lot of money is spent on unnecessary care. Also, we need to free health insurance from employment. It should be like any other insurance.

  16. Lee Parker says:

    Katherine, I stand by my comment. I never said totally in charge. There are times that private and public partnerships work beautifully. Before the changes to FNMA it was a “good to great” company. Deregulation of the Airline industry has been a nightmare. My brother in law is a nurse anesthesis and works for the VA. An ex employee of mine is the director of the physical therapy portion of the VA and yes Katherine and Chris, I have several friends who are doctors and sometimes I have to remind them that I knew them when we were kids (they think they are rock stars).
    Sarah, I don’t get your comment. This is not an all or not proposition and the private portion of the system is to me at a monopoly stage. I am not for 100% socialized medicine but if you think this impending train wreck will stop without intervention, you are dreaming.

  17. Paul PA says:

    It is hard to competitevely price medical care in an emergency (plus who really knows all the questions they should ask). It would seem that a maximum rate for uninsured of say 10% above the average approved/paid by insurance companies could be set. Doesn’t take 1000 pages to do this and most of this problem goes away.

  18. Katherine says:

    Lee Parker, you are dreaming if you think the “private/public partnership” will last more than five years. Because the government insurance program will undercut private insurance on cost, employers will rapidly throw their employee plans away and send their employees to the public “option,” with the result that private insurance will die. Ask yourself why there is no market for private insurance for the over-65s. Medicare plus, yes; but basic insurance, even high-deductible major medical, is simply not available because of the government program.

    FNMA was okay, pretty much, until the government began pushing for subprime mortgages as a “justice” issue for people who couldn’t afford houses. This is another case of government intervention with good intentions producing terrible unintended results.

  19. Lee Parker says:

    Katherine I have a question. I hope this does not happen to you. I pray it doesn’t but if you have a serious illness and you have an individual plan it is not a matter of if you become un-insurable but when (unless you are a very wealthy person). So tell me how you protect the assets that you have worked for your entire life?

  20. Jeremy Bonner says:

    Katherine (#18),

    I would point out that Britain has private health insurance and private medicine (my parents had their hips done that way recently because of the wait time). It doesn’t prove that this will hold true in the US if today’s health care reform goes through, but since the NHS tends to be the whipping boy for the critics of public health care, it’s only reasonable that this fact is also stated. Frankly, the NHS is something of the proverbial curate’s egg – good in parts. It’s ironic that the effort to introduce an internal market over the past twenty years (by Conservative and Labour governments) seems to have brought the worst features of management-driven health care with few of the positive compensations.

  21. Katherine says:

    Lee Parker, I don’t say that the current situation for individual insurance policies is ideal. It isn’t. What I say is that the proposed cure, which is, whether you care to see it or not, a government takeover of medical treatment, is going to be worse than the disease. Obviously any plan to de-link insurance from employment is going to have to take into account people with pre-existing conditions. Your position is, essentially, that rather than a modest fix to the current system which would encourage more choice and make some allowance for situations like yours, the entire system should be changed for everyone in the country. If we’re going to do that, I would prefer changes which would increase competition and choice and not drive more medical practitioners out of the field. Just because “something needs to be done” doesn’t mean that this plan is what we should do.

  22. Ouroboros says:

    No. 12, Sarah, said: “No, as has been pointed out to you and others numerous times—the private system is merely attempting to compensate for the *State system* which the private system is *forced* to support—and all that some keep saying is “let’s throw more State at the problem.””

    This is it in a nutshell. Are those high fees insane? Of course. (They are also never paid, because virtually no insurer simply pays what a provider charges. The insurers have separate contracts with providers and will only pay on certain scales themselves). But they are a result of the fact that hospitals, doctors and nurses literally *could* *not* keep the doors open if all they received were Medicare/Medicaid reimbursements.

    [i] Ad hominem comment delete by elf.[/i]

  23. Lee Parker says:

    Katherine, no one really knows what the plan is. I know its about 1,000 pages so if you have read it please outline for it me. My point all along is “the plan” along with intervention will get us working on some solutions which are truly revolutionary. My understanding of the Swedish “plan” is that it is regulated by government and run by the private sector.

    The big three have a monopoly on health care. We have had laws in the past i.e Sherman Anti- Trust etc. which have dealt with monopolies. I have many friends who are “producers” in society who are struggling mightily with health care. If I have to take a corporate or government job simply over health care availability, that to me is akin to socialization. I am curious as to your insurance provider?

  24. Lee Parker says:

    Ouroboros, you need to go back and read the thread because I did outline my situation of which there thousands if not millions in the similar situations. I’m going to refrain from responding to the personal attack you just made but they usually come from folks who don’t use their real names.

  25. Paul PA says:

    Just pass a law saying people can’t be dropped for getting sick. Make plans available to individuals at the average rate for companies with 2-50 employees. This takes the problem down to preexisting conditions. let people pick up an individual plan at these rates when cobra expires (or sooner) as long as they have an existing plan. let individuals have a tax deduction like businesses.

    Instead of 1000 pages why not just establish broad principles that address the issues on insurance

    however insurance companies just make money on the spread between what they take in and what they pay out. As long as competition exists the only way to lower the cost of insurance is to lower what they pay out ie what the care costs

  26. Lee Parker says:

    Paul PA #24, I think that’s a good start on the insurance side of the equation. I do believe Health Care Providers and the Legal Side must be addressed as well.

  27. Katherine says:

    If no one knows what the plan is, Lee Parker, that’s the worst possible recommendation for going ahead with it. I have not personally downloaded and read the 1000 pages; however, I have read analyses by people who have done so. [url=http://sweetness-light.com/archive/what-the-health-care-bill-actually-says]Here[/url] is one written by a Duke classics professor, John David Lewis. It is rather long, but not as long as the bill. There will be government commissions, without appeal, who will decide what kind of health coverage you can have and ultimately what kinds of procedures will be paid for. Lewis points out, “We may answer one question up front: How will the government will pay for all this? Higher taxes, more borrowing, printing money, cutting payments, or rationing services—there are no other options.” Read it; it’s clear and cites the bill in detail.

    As Greg Griffith pointed out in a comment at Stand Firm, the residual number of uninsurable citizens could be offered a straight government subsidy at far less cost and general disturbance than this scheme, which if enacted will control within a few short years what doctors you can see, what treatments you can receive, and will even delve into your bank accounts and make your tax returns available to the health care officials.

  28. Lee Parker says:

    Katherine let me reiterate, I have never endorsed “the plan” Still, no one has explained it to me yet. The primary reason I am on this thread is to gather data so that I can get a pulse from this audience.. As we speak, I am proactively taking action but mostly what I have heard is rhetoric a multitude of fronts. None of this deals with the 20%-30% increase of most renewals.

  29. Country Doc says:

    The tread on super high medical bills illustrates why people are so confused and are proposing simplistic answers to complex problems. Few, including our leaders, really know how the present system works. Let me, with my 45 years of practice in many fields of medicine show what is really going on behind the scenes.
    First, medical bills are not doctor bills. We are use to lumping this under the term “doctor bills” but the physician’s part is a small fraction of the total, which includes the facility, lab, imaging studies, rehab, etc.
    Second, the amount billed is not the real bill. Most medical facilities and doctors have a contract with Medicare, Medicaid, and the various insurance companies to take what the insurance says is the correct price and the rest is written off. Non-precipitating providers are another story and I will deal with that later.
    For some strange reason a hospital may bill $5000 for an ER visit, but the Blue Cross payment will be only $800 dollars. Medicare may pay $400. That is all that is gotten. Meanwhile the patient gets an itemized bill showing the $5000 and when the insurance company pays, they will get a statement that it all is “paid” even though the $4200 disappears. So why do the facilities bill so much? I have tried to find out and only get vague answers. It seems that they get this price from a company that tells them the “wash” and they some how have to bill for that much to comply with certain regulations. For example, in our hospital, Medicare will pay $3 for a blood drawing and Blue Cross pays about $5 (40% less than Medicare!) However they bill and show a $50 charge! This just inflames patients, just as the $25 Tylenol or $30 Band-Aid. I am told that they must do this. Meanwhile, when the insurance pays, the patient thinks what good insurance they must have and how evil those robber “doctors” are.
    So what happens to the patient with no insurance or “out of network” policy. Dave Ramsey has callers who tell about running up $150,000 in medical care and what are they to do. Well, no one pays that. You can’t get blood out of a turnip. All the hospitals I know have financial counselors. You have to ask for help, and they sit you down and go over the problem. Then they will negotiate a reasonable price, perhaps a payment schedule, and in my experience, even write it all off. Don’t forget, anyone can walk into any ER and be seen and never turned away because of money. In our facility, they can’t even talk money until you are seen. St. Jude Hospital in Memphis takes care of children for free, depending on donations. Most of the Shriner’s hospitals take care of crippled children for free. Most doctors will do this if it is asked for and negotiated. Otherwise, patients and family may be facing collectors if they ignore it all.
    As for the child in the ER that got a plastic surgeon to sew up a mouth wound, well a lot of this is the liability problem. No one wants to do procedures on children. I know of no neurosurgeon that operates on children. This must be done by a PEDIATRIC neurosurgeon. Lacerations, by plastic surgeons, trauma by a board certified trauma surgeons. In the past ordinary doctors did this. In court the first thing asked is “Are you a board certified pediatric—–?” If not, game over. Big bucks.
    Now these super specialists are few and very expensive. They have years of extra training. They know that a significant number of their cases will end up in court for less than perfect results. In most states the statute of limitations does not begin until the patient reaches twenty-one years of age! No wonder OB and other services are so expensive. Our president has said that he would oppose tort reform.
    So what is to be done? The tools of reform are readily available right now. I can solve the problem myself, but I have never been asked! I am not a non-treating physician who is a CEO or administrator of a big Medical Corporation or insurance company. I am not on any government panel, although I was on the White House Conference on Aging under Regan. Again, most of the problems are there because of the intervention already by government. I don’t think the politicians and lawyers will relinquish their hold and power. They will just demand more. It seems that this is what the majority of voters have voted for. Enjoy. Lord have mercy.

  30. Lee Parker says:

    “So what is to be done? The tools of reform are readily available right now. I can solve the problem myself, but I have never been asked!” Country Doc, I’m asking.

  31. Katherine says:

    Here’s a set of recommendations from, of all people, the CEO of Whole Foods, containing many of the common-sense and reasonably priced suggestions I favor: encourage health savings accounts, make individual premiums tax-free, as are employer-paid plans, allow competition across state lines, remove government restrictions on what must be in policies, enact tort reform, and fix the Medicare spending problem. He adds a tax credit for voluntary donations to funds for those who cannot get insurance and cannot pay. There are lots of other suggestions being offered in the public square, suggestions which move us away from government control and towards more personal accountability and responsibility.

  32. Clueless says:

    “So what is to be done? The tools of reform are readily available right now. I can solve the problem myself, but I have never been asked!” Country Doc, I’m asking. ”

    1. Fix the liability issue. Nobody is going to take tough cases or children if a less than perfect outcome means that the doctor loses his house/savings and his kids don’t go to college. If every policeman who failed to protect a homeowner from a criminal, or if every firefighter who failed to keep a house from burning down got sued, fire and police protection would cost a hundred times what it does now, and we would be talking about the “fire care crisis” and how nobody ever comes when you call 911. Indeed, in public housing in big cities like Chicago, where police and firefighters are routinely treated as the “enemy” and where folks are quick to do the “Gates” thing, it is difficult to get anybody to come to a 911 call.

    If you want a neurosurgeon in the ER taking care of your kids brain hemmoraghe after he falls off the dirt bike, you wont get one to come more quickly by threatening lawsuits. Similarly, if you want the ER doc to put 3 stitches into your kid’s face instead of calling in a plastic surgeon, lawsuits for emergencies need to go. There needs to be governmental immunity for all cases seen in the emergency room and binding arbitration with caps on pain and suffering for elective hospital admissions and all clinic visits.

    2. Get rid of doctors for the small stuff. Americans are expected to fill out their Tax forms, they can learn how to manage hypertensive medications, cholesterol medications, diabetes and others. A lot of drugs could be made non-prescription, with nice, clear algorithms as to how to manage them. High school students should be taught to take their BP, check their glucose, and to manage routine medications the same way they used to be taught shop and home ec.

    3. Get rid of much of the regulation that requires licenses for most everything. The only reason that physicians have a monopoly on prescription drugs and procedures is because states know they can make money by licensing and regulating them. Also because folks figure (correctly) that if they stitch up their buddies cut, and it get infected, they will get sued and jailed. People used to stitch up their buddies lacerations, and they still do in Mexico (without anesthesia either). Stop regulating medicine, and stop suing everybody who offers help, and there will be plenty of nurses, med techs, pharmacy assistants and others who would be glad to help you straighten out your medications, give you advice about how to lose weight, treat skin infections, and burns, tape your sprained ankle, pop your shoulder back in place after you dislocate it, and the like. My grandmother acted as village “doctor” in Sri Lanka, and she worked as a music teacher. Her highest training was the equivalent of high school. Her patients did fine. There is no shortage of smart people who could do the equivalent of what nurse practitioners do now, if the costs of running an office were not prohibitive. Those costs for what amounts to a low level urgent care are currently about $120,000 before the practitioner gets paid, and largely is due to malpractice, government regulation and the need to keep up with billing)

    4. Make the patient responsible for his own medical records. We are responsible for keeping our taxes for the last 7 years minimum. We can learn to have a big old folder, or a computerized record with all our information on it. The computerized record can be added to by the doc/nurse/med tech/aunt mildred whomever, when you go see them for whatever. Right now physicians are expected to keep the medical record for 7 years, and the only reason they do is liability. This too is a major cost, and a major hassle, and is a huge problem when changing doctors. In Greece, patients are expected to maintain and to bring their own records with them. In most Asian countries this is also the case. It is only the west who babies patients in this way, mostly because patients are so litiginous that they can’t be trusted to keep the medical record, since it is as much/mostly a legal record.

    5. Stuff under the above care obviously will not be terribly expensive. Aunt Mildred, after she pops your shoulder back in, and scrawls “put shoulder back in socket, and suggested Ibuprofen” in your medical record, isn’t going to charge you 20,000 for the privilege. She will probably charge you 5-20 bucks depending on how much she likes you, and whether Maxine’s boyfriend can do it cheaper. Thus, folks will not need insurance for most medical care. Neither should they get it. Routine care should be paid for out of pocket.

    5. The above will free doctors to take care of the actually sick people who have appendicitis, heart attacks and the like. Such folks will need to have some sort of catastrophic coverage. A cheap insurance plan will cover this, and will be affordable for most people. Most folks under 30 could be covered for less than 1000/year. Most folks under 50 could be covered for less than 3000/year. Most folks under 70 could be covered for 7,000/year, and folks over 70 or those who were already severely ill could be covered if there were some rationing.

    6. And yes. Whatever happens there will have to be some rationing. When more than 50% of costs occur in the last 2 years of life, it is necessary to ask if it is appropriate to offer dialysis to folks over the age of 90. (In England folks over the age of 50 don’t get dialysis). All countries ration care. Some ration rationally, and openly, allowing their citizens to vote on their priorities. Others ration by waiting lists (the Canadian experience) or they ration irrationally (as we do). But all countries ration care. No health plan will succeed until it acknowledges that truth, and decides exactly HOW it wishes to ration care.

    My two cents, brought to you at a steep discount, by the magic of the free market, and the free blogosphere.

  33. DaveG says:

    Don’t forget the costs associated with legislatively mandated coverage for experimental treatments. All it takes is is one complaining constituent and all of us, via higher premium costs, pay for every treatment imaginable, regardless of cost.

  34. Northwest Bob says:

    #28 Country Doc;
    [blockquote] For some strange reason a hospital may bill $5000 for an ER visit, but the Blue Cross payment will be only $800 dollars. Medicare may pay $400. That is all that is gotten. Meanwhile the patient gets an itemized bill showing the $5000 and when the insurance company pays, they will get a statement that it all is “paid” even though the $4200 disappears. [/blockquote] If you have private insurance or medicare or medicaid, these artificially high “list prices” do not affect you. (This includes NW Bob.) But, if you are one of the unfortunate self-pay patients, you are expected to pay the list price! Debt collectors will come after you for all of it. This is the cause of the majority of personal bankruptcies.
    In the old days, everyone was self-pay. You did not see this kind of billing practice. Both of my grandfathers were physicians during the depressions. They often got paid with a chicken or a bushel of apples.

  35. Clueless says:

    One more addition to my comment (32). Every other country that has a national health system pays for medical school. The US requires that physicians go 100,000-300,000 in debt and then be an “independent contractor” to the government. If the present structure of medical school (forced on physicians by the US government after the Flexner report) is to remain, then the US taxpayer is going to have to pay for physician education, just as they pay for police and air force pilots. (With a 7 year payback, like Airforce pilots).

    Alternately they could get out of regulating medicine, ditch the insistance of a batchelors degree, ditch the regulations regarding how much medical “research” each medical school should do in order to qualify for the ability to teach medical students what mainly gets taught free by physicians in the last two years of med school, stop counting the books in the medical library, and go back to the apprenticeship system that worked pretty well for physicians before the Flexner report and cost nothing. (Of course that would require getting rid of minimum wage laws, but hey, they have never applied to physicians). Back in the Depression physicians were trained under the apprenticeship system. After that they went out with no debt and hung up a shingle. With no debt and no costs they could afford to be paid in chickens and potatoes. All the debt, and all the costs can be directly laid at the government’s (and the trial lawyers) doors.

    #33. Those sticker prices are mandated by Medicare. Medicare pays a percentage of what it calls “reasonable and customary fees” and then keeps cutting those. If hospitals know that they will be paid only 10% of the cost of a blanket, and the blanket costs 10 dollars, then they will try to inflate that cost to 100 dollars, in order to be paid 10. Other insurance does the same. This works well for everybody other than the uninsured. They pay full fare. However hospitals and physicians are NOT PERMITTED to charge the uninsured less, or to fail to aggressively collect from them. It is considered MEDICARE FRAUD to write off the debts of the uninsured without going through the process of payment plans/collections etc. The majority of what gets called “medicare fraud” is actually physicians telling joe who lost his job “okay, I’ll just write it off”. That illegal, because you are no longer giving medicare the losest price. If the “reasonable and customary fee” for joe who is uninsured falls to zero, then it is fraud to charge medicare more than zero.

  36. Katherine says:

    I agree with all of Clueless’s suggestions. A huge amount of our medical spending would go down if community clinics and pharmacies were allowed to have pharmacists and nurse-practitioners give medications for minor illnesses and routine conditions, with patients bearing the responsibility for their own health conditions. In Egypt, I can buy thyroid supplements and blood pressure meds we need at the local pharmacy without seeing a doctor, as well as going down there if I have a sinus infection or routine bronchitis and getting antibiotics.

    Insurance should be for unusual medical testing, treatments and surgery.

  37. Lee Parker says:

    #33 NW Bob. You are very right. This group of Americans is growing every day is and is a point I have been trying to make. Many folks who click along every day with employee sponsored health care seem to be oblivious to individuals and small business who are living with poor quality insurance or no insurance at all. I have many stories of this scenario.

    You are also right that the uninsured, indigent or middle class, will pay a retail price which is astronomical for medical care. If not, laws are designed to take everything they own. So you see, there are elements of communism in this scenario.

    Politicians and the American public consistently emphasize that small business is the backbone of our country. If you are a small business owner it seems more like “the system” is trying to break your back.

  38. Sarah1 says:

    RE: “My point all along is “the plan” along with intervention will get us working on some solutions which are truly revolutionary.”

    But that’s just it — it won’t. “The Plan” won’t get us working on some solutions . . . we are heading in entirely the opposite direction of the actual solutions that would work.

    RE: “The big three have a monopoly on health care.”

    I find it ironic that I have no idea who the “big three” in health care are — although I do with cars. Who is this “big three”?

    RE: “None of this deals with the 20%-30% increase of most renewals.”

    “Most renewals,” Lee?

    “Most renewals?”

    Or “renewals of people who have had serious illnesses in the past and cost insurance companies big bucks and now they are determined to make that up?”

    If I have a wreck, my car insurance goes up. In fact . . . I work hard not to have a claim on my car insurance in part for that reason — I don’t want my car insurance to go up. How is it that health insurance companies are supposed to not go up after they have discerned that somebody is highly probable to cost them big bucks? Isn’t that what insurance is? Insuring people against the probability of spending big bucks? And keep in mind — unlike cars and other insurance systems, all of our health insurance [i]Must Pay For All The People Who Need Health Care But Cannot Pay[/i].

    Finally . . . Word to Clueless and Country Doc! ; > )

  39. Lee Parker says:

    “RE: “My point all along is “the plan” along with intervention will get us working on some solutions which are truly revolutionary.”

    But that’s just it—it won’t. “The Plan” won’t get us working on some solutions . . . we are heading in entirely the opposite direction of the actual solutions that would work. ”

    Sarah1, its seems to me that we on this thread are having more honest discussions about this issue from ordinary intelligent people than I have seen in 15 years. The threat of “the plan” is bringing this issue to the table.

    “Or “renewals of people who have had serious illnesses in the past and cost insurance companies big bucks and now they are determined to make that up?”

    What are costs? “Costs” are one of the biggest misrepresentations in the industry. There is no such thing as true costs from an economic perspective. True story of a dear friend. Several years ago he had a heart attack. Not his fault just genetics or whatever. He spent two days in the hospital. Needed stents etc. He was uninsured (bad idea). $85,000. He probably will not recover from this. The “costs” were $42,500 per day.

    “If I have a wreck, my car insurance goes up. In fact . . . I work hard not to have a claim on my car insurance in part for that reason—I don’t want my car insurance to go up. How is it that health insurance companies are supposed to not go up after they have discerned that somebody is highly probable to cost them big bucks? Isn’t that what insurance is? Insuring people against the probability of spending big bucks? And keep in mind—unlike cars and other insurance systems, all of our health insurance Must Pay For All The People Who Need Health Care But Cannot Pay.”

    This system while extremely simplistic is a better model than we have.

    The big three include Health Care Providers, Insurance and the legal community.

    ““None of this deals with the 20%-30% increase of most renewals.”

    “Most renewals,” Lee?”

    “Most renewals?”

    I just got my standard renewal letter for a healthy family of three. 18% increase, so I rounded up on this?

  40. St. Jimbob of the Apokalypse says:

    I guess the big question in all this is : Is it a constitutional obligation of the Government to provide the citizens with health care or, is it a constitutional right to have health care that does not demand significant personal investment?

  41. Clueless says:

    #39 “Is it a constitutional obligation of the Government to provide the citizens with health care or, is it a constitutional right to have health care that does not demand significant personal investment? ”

    Well it is like our “constitutional obligation” of the Government to provide the citizens with educational opportunities that don’t demand significant personal investment. We appear to believe that kids from homes where parents either cannnot or will not monitor and assist with homework, cooperate with teachers, demand respectfulness, obedience, school attendance and homework completion from their children, should somehow have equal outcomes with students whose parents do all the above.

    I think schools would be better if we had tax credits for education, and a standardized exam in all subjects administered every 3 months to make sure kids kept up. Then folks could home school, get Aunt Mildred or Maxine’s boyfriend to tutor, or select public or private schools as they thought fit.

    However, smaller government is not something that we are likely to get to solve either our woeful educational system, let alone our woeful health system.

  42. Mitchell says:

    So let me make sure I understand most of your points.

    1) Even thought the average European lives a longer, healthier life than the average American, and even though all of the polls show the average European is happier with his health care coverage than the average American, looking to Western Europe for guidance on our health care system debate is a huge mistake.
    2) We want the system they have in Mexico and Egypt ??
    3) What we really need is a group of unlicensed people who will dispense medication and assist in routine medical care, with disclaimers and no liability for their action.
    4) If we get rid of Medicare all of our problems will be solved, because the insurance companies will take care of everyone, even though they did not do that before we had Medicare.
    5) If we will stop providing health care to people who cannot afford it they will either die, buy insurance, go away, stay sick but not on our dime, or perhaps some charity will step in an take care of them, but that is not my problem.

    For a slightly more Christian perspective on health care reform, I offer the suggestions of the U. S. Conference of Catholic Bishops.
    http://www.usccb.org/sdwp/national/2009-05-usccb-health-care-statement.pdf

  43. Clueless says:

    #41:
    Even thought the average European lives a longer, healthier life than the average American

    This has been debunked multiple times. Europe counts life span beginning with the first breath, and only once gestation is over, not at birth. Thus all premies who die at birth, aren’t counted. Those who survive to 9 months gestation and come off the vent are counted. Since we have a large number of kids who are premature, and try aggressively to save them all, our numbers look bad.

    Europe has explicit rationing for older people. Over 50 – no dialysis. Over 70 – no back or hip operations. “Take a pain pill” as our president advises. Walk into a US ICU and more than half the folk there will be over the age of 80. Those guys wouldn’t be alive in Europe. However the fact that their lives drop from 82 to 78 doesn’t make Europe look bad the way our preemies whose lives “drop” from 78 to zero. Europe doesn’t count the premies so they come out ahead on the numbers.

    It is not at all clear to me that Europeans love their system. I think those who are healthy do, and those who are not, may think differently. However even if they do, a large part of this is expectations. If one never had any expectation of receiving a cardiac bypass that would let you go back to the golf course at 75 then one doesn’t feel bad that you never received it and simply died at home 5 years early.

    2. The system in Mexico and Egypt (also Cuba and a variety of other places) means that folks get care without breaking the system or going bancrupt. Further this care does not break the system, by having hordes of uninsured people descend on hospitals for strep throats and UTIs because “otherwise they will make me pay if I go to the urgent care”. I do not think it is too much to expect that folks learn to provide for themselves the routine care that they feel they “deserve”.

    A system like Egypt’s means that folks can get their hypertension taken care of, and they can get their strep throat taken care of. The middle class and honest uninsured can get care without going bancrupt or taking off work. The illegal immigrants and undocumented uninsured (those who give a false SSN) can get care without bancrupting the hospitals and driving up costs for everyone else. Right now, not only are costs bancrupting the middle class, but they are also bancrupting physicians and hospitals. Medcaid is the reason that there are no pediatric specialists anywhere outside of Children’s hospitals in big cities. They were run out of business. Medicare is the reason that the specialty “geriatrics’ died. Right now, OB and Neurosurgery is dying. Nobody can afford to give unlimited free care to everybody to wants it. (Not even the Catholic church).

    3. Unlicensed people is what we had before WWI. Licenses and medical schools that required a batchelors came out afterward, in a delayed reaction to the Flexner report, and an even bigger desire to make governments money by licensing folk who had money to buy licenses with. We give our kids tylenol every year, and we don’t have a license. If other drugs were nonprescription (and about 60% of them could be) we could give them those also. In other countries people self prescribe rather than sitting on their fannies waiting for somebody else to take care of them, and whining about how long it takes and how much it costs. I don’t think that folks in Egypt are any smarter than Americans. If they can figure out how to read the instructions, and monitor meds, I think we could also. I teach medical students to do these things. Classes could be set up to teach lay folks also. Much of medicine is not rocket science (some is). Trust me. You’re not as dumb as you look.

    4. Medicare is the problem. The insurance companies have to overbill because medicare grossly underpays. Medicare underpays because medicare pays for routine care. Nobody should get a third party to pay for routine care. If we had a food care provider to make sure we all got three meals a day, breakfast would cost a hundred dollars, and we would spend hours every morning arguing with our food care provider about whether scrambled eggs was a qualified service under our contract, and a reasonable and necessary substitute for gruel.

    If one gets rid of the routine care, then insurance can be spent on what insurance is good for, which is catastrophic care.

    5. If we stop providing health care to people who can’t afford it, they will learn to read the instructions, and provide it themselves, or they will ask a friend/neighbor/son/daughter etc. to assist them with their medications. If they need catastrophic care, (because they break their hip) then insurance is available, and they should buy it. If they can’t buy it, then that government can either give them vouchers for a government sponsored charity hospital, or alternately, folks like the Catholic church could have hospitals that provide it to the indigent. That is what we had before Medicare put the charity hospitals out of business. It worked pretty well, too.

    As to the Catholic bishops, I think (speaking as a Catholic) that those bishops do much better when they speak on theology then when they venture into politics. We might have been spared the Wall street mess if they had stuck to condemning usury as unbiblical, instead of insisting on making sure that loans were accessible to the poor as a “justice issue”.

  44. Joshua 24:15 says:

    Great thread! I’ll add some random thoughts as one of Lee’s evil Big Three (an MD):

    In addition to the Bizarro World pricing that Medicare creates by making illegal any attempt to discount care to the uninsured, one has to consider the horrendous costs that hospitals and providers get to pay for the equipment they need. And, I’m not talking about the latest gold-plated whiz-bang robot surgery system that may or may not add any value whatsoever to your care. I’m talking about nuts-and-bolts items, like a garden-variety CT scanner, or hospital beds, or anesthesia machines and ventilators, or baby warmers, or the eye exam equipment that your ophthalmologist needs to examine you. Just one example from my specialty (anesthesiology): the machine that I use daily to keep people asleep and alive during surgery, which could fit very nicely inside my hall closet, costs substantially more than the BMW I purchased, USED, 6 years ago (yeah, just another greedy, BMW-driving doc…). And our hospital needs 20+ machines to cover all of our operating rooms and other locations. And, that doesn’t count the maintenance contract. A portable ultrasound machine that’s no bigger than my laptop costs 20 times as much. ANYTHING that is labelled “for medical use” has a huge cost-escalation factor built in, and I can assure you that it’s still very expensive, even with volume discounting.

    Medical malpractice insurance: anyone want to guess what MDs in the “high-risk” (i.e., high lawsuit) specialties like OB and neurosurgery get to pay annually, especially in plaintiff attorney-friendly states like PA, or MS?? For these poor souls, $50,000 would be considered a bargain. I count myself very lucky that I have a clean record, enjoy a discount for being in a large group and 10+ years in practice, and live in a state where we have a strong MD-owned mutual insurance provider, so it’s only $20,000 off the top for me. That cost has to get passed on somewhere. For the President and his allies to talk about comprehensive reform without targeting tort reform is laughable, and dishonest. One can talk till the cows come home about “improving quality and outcomes,” which any sane MD would naturally support, but you’ve still got a whole lot of hungry lawyers making boodles of $$ off of the system, and generating a whole lot of “defensive medicine.”

    Others have spoken to the notion of “cost shifting.” Essentially, private insurance has traditionally been seen as an escape valve for the underpayment by Medicare/Medicaid. As an example, Medicare pays my specialty 30% of what the Blues pay, and that’s factoring in negotiated rates, not “non-participating” rates. If you work in a community with a large Medicare-Medicaid patient population, as I do, you notice it. Clueless has eloquently described why there are few pediatric specialists outside of large children’s hospitals–it’s Medicaid. At our peds hospital, virtually ALL the specialists on staff are subsidized to one degree or another, otherwise they couldn’t afford to stay in practice.

    I’d also echo what Clueless says about European longevity numbers and rationing of care. That said, one of the many serious questions that Americans need to debate, in a reasoned manner (if that’s possible in this hyper-partisan age), is what we’re going to be willing to pay for in terms of health care. There is a point at which throwing more money at “health,” like any other problem or sector, becomes simply wasteful. I adamantly oppose euthanasia and the notion of so-called “death panels,” but it IS a fact that the lion’s share of $$ are currently spent on the last six months of life–and I think that even the most pro-life MD or nurse will tell you that there are many, many people with very unrealistic notions of “the best possible care” for their aged loved ones, and what results is often far more painful, expensive, and undignified than what might hold with a better acceptance of the inevitability of death, and more emphasis on good palliative/hospice care, not “death by ICU.”

  45. Lee Parker says:

    # 43 Josua, it is important that you realize that I have not characterized physicians as evil. I do characterize the profession as one of the most intelligent grouping of folks on the planet but one of the most arrogant.

    I truly believe that until physicians admit that they are part of the problem and get out of the defensive mode, we will never begin to have a solution. I have never heard a doc say ” you know what this is a mess but I am grateful that there is an entire industry (insurance) out there designed originally just to pay me and for my ancillary services.” I believe that this industry (insurance) has morphed out of control and I may be wrong but is there another country in the world where a physician can make 000,000,000 net income (I’m not saying undeservedly) regardless of a $50,000 insurance premium.

  46. Clueless says:

    #43- I believe that this industry (insurance) has morphed out of control and I may be wrong but is there another country in the world where a physician can make 000,000,000 net income (I’m not saying undeservedly) regardless of a $50,000 insurance premium.

    Those are relatively few physicians who earn more than 200,000. The majority earns about 100,000 for a 60-90 hour work week. Once one factors in the inability to earn more than minimum wage until age 30-35, and the cost of medical education, most American physicians earn less than American plumbers over a lifetime. Not that we are poor. We aren’t. We are wealthy. However, so are plumbers, electricians, lawyers, accountants, and small business owners of various types whose hours are usually fewer.

    Counting in retirement benefits, American physicians make less over a lifetime than do Chicago police officers and firemen.

    Either way, the physician salary component is less than 10% of medical costs in the US, and even if physicians worked for nothing would not come close to solving the health care crisis.

    In point of fact, everybody in America is overpaid compared to those outside of America. Saying that Indian physicians make about 50,000 (US) dollars a year (which they do) and so that should be what US physicians should make, ignores the fact that cooks and maids in India make 500 (US) dollars a year. Indian physians live like kings. The standard of living is much lower. That is why many of my friends who originally came to the US are now returning back to their native country where they can earn less, and live better. (Some of them, who plan to focus on medial tourists from the US) will earn more, not less).

  47. Lee Parker says:

    “Counting in retirement benefits, American physicians make less over a lifetime than do Chicago police officers and firemen.” Clueless, I spend too much time with business associates, friends and family who are Physicians, Dentists etc. and I disagree with you in the strongest of terms.

    Having said that have you ever done any plumbing? It is nasty, very tough on your body and there is little if any satisfaction relative to being a healer.

    On another topic has anyone studied the Cleveland Clinic Model?

  48. Clueless says:

    Well, I’m a neurologist, and neurologists make more than internists, family physicians and pediatricians, but less than surgeons.

    My salary was confined to summers 1,500/year through college and medical school 8 years after high school (I left with only modest debt thanks to generous parents).

    During my intern year 1981 I received 14,000 annually before taxes for a 110 hour work week.

    During residency and fellowship (5 years) I received between 17,000 to 21,000 annually before taxes for a 90 hour work week.

    Then I received 30,000 for 4 years 1987-1991, and 70,000 eventually increasing to 81,000 before taxes by 1999 for an 80 hour workweek in academic medicine.

    1999 to 2009 I was in private practice. My earnings have bounced around but my total earnings before taxes averaged 160,000 for a 110 hour work week.

    More recently I took a 30% pay cut (before taxes) to work for the VA and to got a 50 hour work week.

    Not bad, and I don’t complain. I agree that I’m rich.

    However, my neighbor accross the street owns a small auto repair shop. He had no educational debt, began working after high school and took a 80,000 debt for the shop. He makes over 100,000 before taxes. He works about 60 hours/week.

    My other neighbor accross the street owns a small hair salon. She also has no educational debt, has worked since high school also works about 60 hours and makes about 50,000 before taxes.

    My next door neighbor works for the city in the Dept. of Transportation, fixing roads and stuff. He too has no educational debt, has worked for the city since high school, and he makes 60,000 before taxes, and works a flat 40 hour week

    All four of us have similar homes, cars, and standards of living. Our kids go to the same schools and have similar extracurricular/churhc type activities. And that’s fine by me.

    So let us hear it. Currently medical student graduate an average of 150,000 in debt. How much is “fair” for a US physician to earn? And how much do you earn?

  49. Clueless says:

    “[Plumbing] It is nasty, very tough on your body and there is little if any satisfaction relative to being a healer.”

    As to the satisfaction of being a healer, the drawbacks of dealing with insurance companies and government regulations, while worrying about lawsuits have greatly diminished that satisfaction.

    I rather like the idea of plumbing, though I would prefer electrical work if I had to do it again. There is always the satisfaction of a job well done in the blue collar trades (and colonoscopies are just as “nasty” as plugged sewer lines). I would advise any son of mine to consider plumbing. (It does require brute strength as well as intellect).

    I have already advised both my kids not to go into medicine. I don’t know any physician who has advised his kids to go into medicine recently.

  50. Mitchell says:

    I am self employed. I run a small consulting business, normally work about 80 hours a week. Make about $110,000. I have 5 employees. I have a masters degree in accounting and a doctorate. I also graduated with little debt due to my parents. First job out of school paid about 21k per year. Last year between my health insurance and the health insurance I pay for my employees I paid approximately $26,000, plus I paid out medical bills and drugs bills totaling $7,500, I have a high deductible plan and my wife had a rather serious health issue. Thus in one year I paid about $33,000 into the health care system. Our insurance costs have been going up at the rate of about 10% per year. Small business is being killed by the cost of health insurance.

  51. Clueless says:

    Shall we agree that 110,000 for 80 hour weeks is “fair” after costs of an education through the doctoral level, and “paying one’s dues” at minimum wage for 10+ years after school ?

    If so, most internists, pediatricians and family practice physicians are severely underpaid as they work considerably more than 80 hours for slightly less pay.

    Or is your contention that you also are paid “too much”?

  52. Lee Parker says:

    Clueless, I hope that you realize that I nor any one else has challenged you on what you are paid. Amen, Mitchell your situation is slightly better than mine. I believe there are many micro businesses in he market that are getting tattooed by the health care system. It is my belief that we should have the same access and pricing that employees of Blue Cross and Blue Shield have.

    It’s a much bigger problem; however, when preimiums are rising as much as 30% at renewals. I still contend that the combination of Health Care Providers, Lawyers and Insurance Companies will not and have no incentive to solve the problem. I have not heard anything that changes my opinion to this on this thread.

  53. Clueless says:

    I have no doubt that numerous businesses are getting hammered by the health care system. When I was in private practice our biggest problem was covering our own employees (I was in a group of 4 physicians and we had 12 employees, some of whom had significant medical expenses). Physicians are small business owners also.

    The issue is how best to solve the problem. Currently the game plan appears to be simply reducing payments by Medicare/Medicaid further to physicians and hospitals, meaning that insurance premiums will continue to rise higher. It is not possible to significantly reduce physicians pay without simply driving them out of business. When I was in private practice, I found that I had to work more than 60 hours/week just to pay my overhead, before I earned a dime. Increasing my overhead (by making me pay for a computerized medical record) and decreasing my payments will not help matters. I would be forced to simply refuse to see children, medicaid, medicare and ER consults. (That is what many physicians have chosen to do, and it does increase their pay, possibly Lee Parker hangs around them). There is a limit to how many hours any person can work, and I found that I had reached that limit earlier this year, which is why I changed jobs to a government one, where I no longer have to deal with medicaid, medicare or the uninsured.

    But as I said before, even if physicians continued to work 100+hours and never made more than I made as an intern (14,000) it would not solve the health care problem, because as I said, only 10% of the cost of health care is generated by physician salaries.

    I think it would make more sense to reduce the costs associated with practice, rather than demonizing physicians. Getting rid of the burdens of malpractice and regulation, and keeping up with byzantine insurance schemes and maintaining the medical record would greatly reduce costs. So would ending the monopoly that physicians have on prescription drugs, and empowering patients to treat themselves for most illnesses.

    None of these items, however, which would actually lower costs are being proposed.

  54. Mitchell says:

    #50 I am not complaining about my pay. Nor have I ever complained about the pay of physicians. I have worked hard and continue to work hard as do they. The cost I pay for health insurance does however prevent me from expanding my business and creating new jobs. I could create two jobs for what I am paying for health care. I could stop paying for the health insurance for my employees, but that would require a pay cut for them and at least two could not obtain private policies because of preexisting conditions. Small business simply needs a less expensive option.

  55. Lee Parker says:

    Clueless, please just humor me on this and get out of the defensive mode. Let me say this about the income side and I’m done. Without divulging confidentiality, of the medical folks I am associated with, none make less than $200,000, at least two make $500,000 and one I suspect makes $1,200,000 or so. Several work extremely hard and several do not.

    Who has demonized physicians on this thread? If it was me I apologize. Have you studied the Cleveland Clinic Model as the administration is very interested in it and my father says it is the best hospital in the world.

  56. Clueless says:

    The Cleveland Clinic is basically an academic medicine model. It works as long as there is a steady stream of underpaid interns to maintain the workload of the attendings to under 80h/week. (I was paid about 71k-81k in academic medicine, and it was not too bad, given the improved hours. Unfortunately, fewer interns (even from India) are entering the teaching hospitals (and they now limit intern work to 80h/week), and physician assistants and nurse practitioners expect to be paid 60k for a 40h work week. Thus, attending workloads have been increasing and more and more attendings are deciding they have had enough.

    Thus, even the academic hospitals are moaning about how they are unable to survive (despite a state subsidy in most cases). Most of them are in the process of changing their compensation practices to a private practice model because they can’t stay in business, and considering ways to limit their medicaid population.

    The bottom line is that it is starting to be difficult to find somebody to cross subsidize medicaid and medicare. I imagine that the physicians you deal with are in either plastic surgery, pain medicine, cardiology or orthopedics or that they refuse to see kids, medicaid, uninsured or medicare.

    One can indeed make a very good living if one refuses to see government pay folks. However I think most docs try to pull their weight. Nobody went into medicine thinking about turning their backs on the poor.

  57. Clueless says:

    “I could stop paying for the health insurance for my employees, but that would require a pay cut for them and at least two could not obtain private policies because of preexisting conditions. Small business simply needs a less expensive option. ”

    Catastrophic coverage and the ability to write one’s own routine care prescriptions would certainly lower the price.

  58. Clueless says:

    #51 “It’s a much bigger problem; however, when preimiums are rising as much as 30% at renewals. I still contend that the combination of Health Care Providers, Lawyers and Insurance Companies will not and have no incentive to solve the problem. I have not heard anything that changes my opinion to this on this thread. ”

    What I resent is the implication that physicians have much control over this. Medicaid and Medicare set our rates. Our only option is to refuse to see such patients (and we hesitate to do so because they are a majority, and they tend to be the ones who need us most). The government and the insurers require that we maintain hospital privileges, and the hospitals require that we see all comers in the ER for free. Virtually all hospital call is uncompensated, and that was 30% of my practice (usually done in the middle of the night). In order to make sense of the byzantine regulations that medicare and insurance has arranged for us, we need a huge staff to keep us from committing “fraud” by calling something a level 5 when according to the government, it should be a level 2 because you forgot to ask the comatose patient what his family history was, while spending 3 hours, trying to treat his meningitis in the middle of the night.

    We have no control over any of these things. Yet we are the folks who are on the front line, and we are the ones blamed because the “system” is broken. Yet, we are not even invited to the table where more regulations will be devised to further burden us, and ensnare us.

    So yes, I think that physicians are being demonized.

  59. Country Doc says:

    Cluless speaketh true sooth! I affirm everything he says. He has been there and so have I. There is an army of highly trained physicians out there that know where the bodies are buried and what the answer should be and are never invited to the table. Why? Well because it is not about better care and controlling costs. It is another power grab and Big Government controlling every aspect of the serf’s lives. Some just call it Marxism. Those of you who think that out of DC will come a plan that will deliver quality care and the bill will be sent to someone else are going to be really angry, but there is nothing you will be able to do about it. Enjoy.

  60. Lee Parker says:

    “I imagine that the physicians you deal with are in either plastic surgery, pain medicine, cardiology or orthopedics or that they refuse to see kids, medicaid, uninsured or medicare.” You nailed that one, Clueless, except one is a regular old dentist.

    So if we subsidized Mayo, Cleveland, Duke, etc and absolutely ruled out frivolous tests could we deal with catastrophic situations more efficiently? You can’t sue me if I work for a quasi government institution. Because I suspect you are very competent and I know some very competent health care professionals who work for the VA in my town would you see me for the ailments I just couldn’t stand anymore? i.e.. ear aches, broken toes etc? What would my insurance premium look like for this care or could it be so reasonable that I don’t need insurance?

    On another note, why does the medical profession require a “hazing” type of initiation to become a MD. Frankly I’m more interested in your ability to practice more than I am how long you can stay awake. There is a shortage of Md’s in this country. Is it artificial?

    This is the part of your profession that I wish that Docs would take responsibility for.

  61. Lee Parker says:

    Country Doc in a crisis sometimes you don’t wait to be invited to the table. You just walk up and turn it over.

  62. Country Doc says:

    Lee, as for the hazing, I think medicine is so hard. There is a tremendous amount of data to down load into the students brain and skills that must be learned that it would be impossible to shorten or make easier. Training is getting longer and longer. We call that residency and then fellowships which seem to add years more all the time. Now for sure, much of what students, residents and some of fellowships do is “scut work” or cheap labor. The university hospitals would close if they had to hire and pay proper wages to replace all the labor done by students, etc. There is now a cap of 80 hours of duty in a week—maybe. The schools are really trying to get the rule changed back to the old rule which was that you could not work more that 24 hours in a day! I don’t know how to increase the number of doctors, since it depends on suitable applicants and more medical schools. Where will the money come from to build them? You can’t believe how much any medical facility costs. As for turning over the table, well there shouldn’t even be a table. The problems are due to the legal profession and government. People who try that end up in jail. Our way to survive is simply to withdraw and not participate. Just look at the early retirement, non-perticipation in Medicare/Medicaid, HMO, etc. It will accelerate. I was chosen by our right wing governor to be on the White Conference on Aging put on by the Clintons. There were 2400 delegates–most were government people, union heads, Native Americans, Gray Panthers, and even a delegation on equal rights for homosexual elderly! There were twelve physicians–six were geriatricians which included me. I went ot all the committees and made great input, all of which was ignored. The final recommendations were more socialism and Hillary care. It was a useless exercise. The fastest area of growth in medicine is the over fifty-five physicians going to the mission field. It looks interesting.

  63. Clueless says:

    “So if we subsidized Mayo, Cleveland, Duke, etc and absolutely ruled out frivolous tests could we deal with catastrophic situations more efficiently”

    The government already subsidizes the academic hospitals by supposedly paying them for “teaching” interns and residents. The government pays some 100,000/houseofficer despite the fact that the intern gets paid very little, and is a lot cheaper than a nurse practitioner. This subsidy is BECAUSE academic hospitals see sicker patients and have a higher Medicaid/medicare population.

    If one simply allows the regular hospitals to die, and to only have academic hospitals, (was that your suggestion?) then yes, costs would decrease because waiting lists and travel time would increase. That has been proven with specialty pediatric care which has gotten so scarce (it’s mostly medicaid) that kids have to drive around 300 miles and wait around 3 months for appointments. If the elderly also had to drive 300 miles and wait 3 months before going to the academic center, certainly costs would come down. It is known as rationing by access, and is the way Canada rations. Half the folks wouldn’t make the drive and the other half wouldn’t survive the wait. Routine care is unavailable and catastrophic care is indeed “more efficient” as you put it.

    “would you see me for the ailments I just couldn’t stand anymore? i.e.. ear aches, broken toes etc? What would my insurance premium look like for this care or could it be so reasonable that I don’t need insurance”

    Unless you are a veteran I am not permitted to see you officially for the VA, nor may I see you in a private hospital. I am permitted to see you in a free clinic setting.

    “On another note, why does the medical profession require a “hazing” type of initiation to become a MD. Frankly I’m more interested in your ability to practice more than I am how long you can stay awake. There is a shortage of Md’s in this country. Is it artificial?”

    The hazing is a left over from the charity hospitals. The old system (before Medicare) involved teaching physicians their trade using charity hospitals who didn’t charge patients. Physicians were called “house officers” because they did not get paid, but lived in the hospital for free, got free meals from the hospital kitchen, and the sisters who ran the places would do their laundry along with the patient’s linens. The idea was that all the house officers time belonged to the hospital to be spent taking care of patients, and he/she would be compensated by the education he got, practicing on those patients. (Patients and drug manufacturers would also often give ice creams, candy or basball tickets). The attending staff checked in once a week or so to see how the interns were getting along (they weren’t paid either).

    The arrival of Medicare and later Medicaid meant the charity hospitals shut down, and training of physicians was shunted to private hospitals that previously were the domain of the attendings. However the private patients did not wish interns practicing on them, since they were after all, paying. Medicaid patients also decided that they too didn’t want no interns practicing on them, even though they weren’t paying, and nor was the government paying cost, even then. Thus, the rules regarding house staff supervision came in, and have tightened every few years such that house staff now are very tightly supervised by attendings, who have to see their patients every day.

    However, there was still not a lot of extra money to pay the interns/residents, nor was there a tradition of paying them. Medicaid never paid costs, and the presumption was that house staff shouldn’t be paid since they were still in training. It was still expected that all their time belonged to the hospital and that they shouldn’t be compensated (although as medical school tuitions rose, compensation for interns began, and has increased since then, though it still compares unfavorably with the compensation of Wal-Mart greeters). Also, after the well publicized death of Libby Zion, resident work hours were limited to 80 hours/week a few years ago.

    So the “hazing” of young physicians is not really hazing. It is the way medicine took care of the poor then, and continues to be the way medicine takes care of the poor now. However the difference is that now, in addition to underpayments while in training, physicians as attendings, are now also paid less than cost for seeing medicaid and medicare, and are also required to see patients in the emergency room whether or not they can pay, if they hold hospital privileges. The medicaid/medicare payments being held under cost came in in 1981 with DRGs, and the stipulation that we show up in the ER regardless of the hour, and regardless of if we get paid came in with EMTALA a few years ago.

    So no. That is not a part of our profession that docs should take responsibility for. It is another of the patchwork by which care was initially provided to the poor and the sick, which has been taken advantage of by the Federal government. The physician shortage is real, and is global. In point of fact, there is a shortage of people who wish to go 150,000 in debt and to work 100 hours a week in order to get paid less than an auto mechanic. I don’t know why that should be surprising. Most people who have 150,000 and are willing to work hard, could open up a Panera bread franchize and get paid more. An endocrinologist I know did that (opening a bagel place near the hospital, and taking the early morning shift before 8am rounds, after which his wife worked until closing). He eventually was able to retire.

    “Country Doc in a crisis sometimes you don’t wait to be invited to the table. You just walk up and turn it over. ”

    Only physicians aren’t allowed to strike (it is considered a RICCO offense). We are not even permitted to ask our collegue in the doctors lounge what they charge, (that is illegal price fixing and restraint of trade). All we are allowed to do is lobby (as long as we do it nicely).

    Physicians do not enjoy the freedoms given to lawyers, accountants, or blue collar workers.

  64. Lee Parker says:

    This is an awesome education and thank you for the caliber of responses Clueless and Country Doc. I’ll be out of pocket for a couple of days but please hang around as this thread has much more depth to it and I truly believe it to be helpful. I still believe that the fix will be a political one, it’s all we have left, and it is radically different than “the plan.” It still looks like a “two tired” system.

    Country Doc, I know its asking a lot but would you outline your fix and would you comment Clueless? Thanks!

  65. Clueless says:

    “would you see me for the ailments I just couldn’t stand anymore? i.e.. ear aches, broken toes etc? What would my insurance premium look like for this care or could it be so reasonable that I don’t need insurance”

    To continue the theme of self prescribing, you can learn to do this yourself. The ear ache aint hard. Somebody needs to look in your ear, there are pictures on the internet of normal ears and infected ears, and otoscopes are available at Wal-greens. My guess is that this duty will be defered to the pharmacists who will be allowed to prescribe, assuming everybody is not eventually allowed to prescribe.

    As to your feet, I would get “Office Orthopedics for the Primary Care Practitioner”. It is a skinny book with large pictures and very simple writing. It will tell you how to evaluate the joints, long bones, muscles etc.

    Briefly, assuming that somebody has done an X-ray and your toes are not in fact currently fractured, and there is no infection of the bone,, the question is, is this a problem of the joints (palpate them, wiggle them and find out), is this a problem of the tendon, pulling on the ends see if it gets better with a strap around the arch, temporarily, or a heel spur (same deal). Once you have figured out where the problem lies, the book will suggest exercises, braces, prosthetic shoes etc.

    If you need prosthetic shoes (often necessary with broken toes) then you can try to rig up one yourself. Even if it doesn’t work, it will at least tell you what does and doesn’t work for you, and will make it easier for the professional shoe maker to make a pair that fits you. If you have had broken toes, now healed but still painful, you will need a hard sole (or else the soft shoe of a sneaker will act as a lever aganst the injury as you walk. Since you will be adding stuff,you will need a slightly larger shoe than you would normally get. If you have myofascial pain you will need to build up the arch with plastic strips and duct tape. If you have heel spurs, you will need to add a “donut” that has a depression into which your heel spur wll fit. Then you will need to fill in the sides, so the shoe does not fall off as you walk.

    This is what physicians, podiatrists and shoe makers do. They figure it out. They practice medicine until they get it right. You can do. It’s not rocket science.

  66. Clueless says:

    As to setting up new medical schools, it happens that I know something about this also. The local state university is setting up a Campus in my hometown, as they simply have no space or manpower to expand their campus in the capitol city. Thus, this year six fresh faced 3rd year medical students are here to learn how to be docs, next year we will have 3rd and 4th years and eventually the plan is to have 60 kids.

    The medical school branch has 4 people who do administrative stuff, mostly funding and making sure the school gets certified. This is sufficiently expensive thanks to government regulations, and the price of licencing etc. that even though all 6 kids are paying 40,000 year, they are over 15 million in the hole. A bunch of local businesses have donated the money. They figure that they will need 45 million eventually to make it work.

    The 4 administrators get paid (the chancellor 150,000, the rest 30,000 to 60,000).

    None of the physicians who do the actual day to day teaching get paid anything. We do it free. (Physicians have always taught medical students and residents free, even in academic medicine, the doctors salary comes from the private patients he sees, and from research grants, not from the students or the school).

    The attendings are all volunteers (since we are all private practice or government). I personally have the kids with me 3 half days/week, and I have lectures/quizzes/case conferences (which I prepare) according to a syllabus (which I also prepare) one day a week. It is a fair amount of work, but I like teaching.

    However, notice how much the medical students are paying in order to have me and 12 other docs teach them for free. Forty grand a year.

    Notice also how much a medical school costs even though the medical students are paying huge, and the physician teachers are working for free. Forty five million a year. Most of that goes to support government bureacrats who will be flying back and forth to count the number of books in the library, and to interview me ad nauseaum, and to look over the materials I prepare, etc.

    So yes. The doctor shortage could be alleviated. But more government regulation will not do it.

  67. robroy says:

    Country Doc is correct. The medical bill is all phoney nonsense. My most expensive operation is a cochlear implant. One of my patients (made a mistake and) looked at her bill. It was for something like $100,000. But the cost of the implant is $20,000 and the hospital probably gets paid $23,000 for it (Medicaid actually pays the hospital $13,000 so the hospital eats the loss). One of the hospitals here was going to tell me to stop because it pays so little. I think that I get somewhere in the range of $1500 for it. It’s a 3 hour procedure where I have a burr going 50,000 rpm right on the facial nerve – a slip would paralyze the patient’s face. Of course, that $1500 goes in the kitty, and I pay all my employees with that money. So what’s my cut? I don’t know, but it ain’t much.

    There is one real ramification of the silly medical bills. I would love to be able to charge an uninsured patient $25 bucks or whatever they can afford for my services. But if I charge them less than Medicaid, that is considered Medicaid fraud. So I bill them a ridiculous amount knowing they can’t pay and write it off when they don’t.

    This brings up a great idea which go a long way to help take care of the uninsured. Let physicians write off the free care that we do. I do a lot of free care because I am a fool. I do more than the others in town. I am not complaining. I do it voluntarily – it is some of my most rewarding work – but there is much more work than I can do by myself. If physicians could take it off their taxes, a lot more physicians would be volunteering.

  68. Jeremy Bonner says:

    Robroy,

    That’s one of the most practical suggestions on this topic that I’ve heard in a while. It’s also a far more legitimate tax write-off than many “business expense” claims that people currently make. Have you ever proposed it either to your political representatives or to the medical associations to which you belong?

  69. Country Doc says:

    Tax write offs have been suggested to federal and state legislatures. They always turn it down. Guess they just can’t see not taxing to the max.

  70. Lee Parker says:

    Robroy, absolutely no problem with your fee. Do you have a good understanding of why the hospital is $20,000+?

  71. Country Doc says:

    As I pointed out, and Robroy mentions, this is the a la carte price list derived fron the “wash” and not related to reality. Granted, a large part of it is for the implant cost which they have to pay and have no controll over. However, the real price will be set by the insurance company agreement or Medicare/Medicaid. As for free care, we all like to do it but the liability remains, and we can’t give away the lab, xray, implants, medication. That is controlled by others. I can do free sleep study interpretations, but that is useless if there is not free sleep lab service and free CPAP machine provided after it is all over. Lot’s of retirees would like to work in free clinics but can’t afford to keep up their liability insurance. This goes for churches, etc. offering free clinics. Tort/court/legal reform is the first thing that has to be done or there will be no cost control on health care.

  72. robroy says:

    Lee, the implant cost is ~$20,000 which the hospital pays for. Hence, the hospital ain’t making much if they get $23,000 total. Is the device worth $20,000? You bet. It cures the deaf. It is amazing. Could it be sold for less? The medical device manufacturers have been hit with big lawsuits. One of the three cochlear implant marketers had problems with meningitis post-operatively. They were on top of things and figured out what the problem was which was only discovered in post-marketing surveillance but they still got hit by lawsuits. These devices are made for a very small market. So I think the price is about right.

  73. Lee Parker says:

    Help me out with another scenario. We have have a hospital in town which performs procedure after procedure relative to bypass, stents caths etc. The surgeons and technical folks have been doing it for so long it has become almost “simple” for them in terms of protocol etc. I’m guessing but the average soup to nuts delivery of a patient is something like 4 days and the average cost is around $100,000 or $25,000 per day.

    If one goes to what is probably the “best” heart hospitals in the world (Cleveland Clinic) about the only difference in “costs” is the plane trip. If there are truly market forces at work here why are the costs so similar and why is that “routine” surgery $100,000 in the first place?

  74. Country Doc says:

    Lee, let me say this again. The price is dictated by the government and will be similar in all places. What is the true bill is what they pay, which will be a fraction of that $100,000 you quote. Now if you want to do it cheap and good then try medical tourism to maybe Beleize, Panama, Costa Rico, India where it will be done by US trained specialist in a Marriott or Hyatt hospital, one RN per patient and two weeks vacation afterward in their resort all inclusive including airfare—$10,000 to $20,000. Now why is that? Lots of reason. One is no lawsuits, cheap labor, and all private.

  75. Lee Parker says:

    Thanks Country Doc. I have another question and I’ve touched on it a bit. Do you know anything about the Congressional Health Insurance Plan and how it works. I really know nothing.

  76. Country Doc says:

    As far as I know, it is a super dupper health insurance for Congress and their families, even after they retire or aren’t re-elected that covers everything with no rationing and very low co-pay. What a deal, but not for the peasants. Most federal health plans for federal workers are also very good. That’s why the congress members and President are asked if they will be interested in dropping their plan now and enrolling in the one they are cooking up for the rest of us. Usually they just don’t answer. I think the real answer is that coming up with a good insurance plan can be done by the authorities, but they just don’t want to pay for it. Maybe complete health care is too expensive for the ordinary person and we just need to acknowlege it. The main problem with socialized medicine is not only the ineffeciency of managment like most government activities, but simply that they want to do it on the cheap and cut corners, while claiming to give it all to the voters.

  77. Country Doc says:

    Lee, you asked for my plan. This is it in outline form. I hope it is not too long for the thread. Now you use your influence in getting Rob Roy, Clueless, me, and maybe Dr. Bluemberg of Fox to be on a blue ribbon team to go to DC, first class and stay in a five star hotel eith limo to carry us around. I think could get it all done in about a week. Cheers
    After 45 years of medical practice across several specialties and various socio-economic groups these are my recommendations. These are all doable now and would not raise the cost or decrease the quality of care. The number one way to ration care is to limit the number of doctors who practice or participate in any scheme. Each of these points is in sketchy outline form, and each could be an entire chapter.
    1. Tort reform. This is the lynch pin of reform. Without it no other plan can work. I estimate that 35% of the orders I write are due to patient and family demands. I am not going to have some shyster lawyer asking me in one of our corrupt courts why I didn’t order it.
    2. Uncouple health insurance from employment. Just give everyone a tax break for insurance cost. Let the patient own the policy and not the employer.
    3. With the above, encourage group buying of insurance. This can be a trade group, professional organization, club, church, co-op or any other bargaining group.
    4. Sovereign immunity for emergency care and free clinics.
    5. Tax credit for charity care.
    6. Encourage health savings accounts. This could even be done for Medicaid patients. Give them a debit card to pay the bills and when the year is over, let them transfer a percentage of the money to an IRA type account or educational account for their children. Now they would have an interest in saving money.
    7. Encourage self care. Not just good health practices, but how to treat themselves with over the counter medicine or pharmacist advice. The Agricultural Department use to put out great paper booklets on every aspect of farming and gardening and food preparation. You could get a book on building a chicken house or canning food. There could be free, such booklets on various diseases. Make them free by making a deal for super markets stocking them.
    8. I would prefer no government involvement in health providing. I say get back to the Constitution such as Senator Dr. Ron Paul is preaching. That being said, if the state is going to be the big fish in the pond, then they need to simply set a fee for all services they will provide and let the providers choose who they will treat for that amount and let them selectively set a higher price if they desire, but have the set fee sent directly to the provider and let them bill for the difference. Then patients and doctors would have freedom of selection without penalty.
    9. Use government money to pay teaching centers such as medical schools an amount for the total number of patients they see each year. Medical school clinics, charity hospitals, Federal health clinics would then have to depend on care to fund their budget. They could have a sliding scale fee based on patient income. This could be from zero for the destitute to full pay for the rich. Education for doctors would be boosted with these patients available to treat and be taught by fully trained teachers. This was what we had before Medicare/Medicaid and it worked if adequately funded. This would simplify the annual budget lobbying of the legislature for funds. It the school needed more money, then bring in more patients.
    10. For drug costs, have a plan that paid 100% for generics and certain over the counter drugs and a list of special non-generic and the price that would be paid for them If the pharmaceutical companies wanted more than the plan price, then they would have to get if from the patient. Patients then would shop and get their physician to favor the cheaper drug. This should bring down prices. Or the drug companies could negotiate the price to get their drug on the list. This is better than the 100 or so plans that cause doctors to give up on what is on the patient’s plan. This would not prevent insurance companies from coming up with their own panel of drugs, but they would be up against the basic preferred plan.
    I’m sure there are things that would need fine tuning, but I believe the above is sane and doable now. Those of us in the trenches and with training and experience should be setting these policies and not politicians who can’t seem to manage their own lives. This is a start.

  78. Country Doc says:

    I think I meant Dr. Rosenblume of Fox news.

  79. Lee Parker says:

    This is a great start. Thanks Contry Doc.

  80. Lee Parker says:

    Coutry Doc and Clueless, the following are comments based upon a cursory review of yours. This is a highly competent individual with great experience in health care. Before I tell you about the background of this individual I would like to here what you two have to say.

    “Your physician friend’s reform plan is physician centered and I’m not surprised since they are sort of the center of healthcare. I would hope that it would be more patient focused. My concern is that somewhere along the way, we lost the ethic of the professional and the ideas of business took control. I suggest A Second Opinion, Rescuing American Healthcare by Dr. Arnold Relman, former editor of the New England Journal of Medicine.

    My comments follow:

    1. I’m in favor of uncoupling health insurance from employment. There would need to be some serious modification of current insurance regulations to manage the issues of adverse selection. This could be done with revisions of the how plans deal with risk—such as reinsurance. I’m just not sure that the model where companies profit from not paying for sick care is the best way to finance caring for the sick and promoting health. I truly don’t see the logic of this.

    2. Tax deduction rather than tax credit for professionals providing bona fide charity care. But this would open small private practices to routine audits from IRS rather than Medicare. Be careful what you wish for. The state of SC could actually do this. It would then allow physicians to begin accounting for their charity care. Most physicians actually do less than they think, their staffs are very good at constructing barriers to access. Most physicians get their charity cases from ER/hospital admissions they get and existing patients who become uninsured and not from people trying to get an appointment to come through the front door. I know because I’ve audited the practices to do their revenue forecasts. Most doctor’s actually don’t have a good idea of who they treat and how much surgery they do. They always estimate more than they actually do. They are busy and it is easy to translate busy with productive and many many patients.

    3. HSAs are OK but they only work well for higher income young professionals. I’m not certain insurance is the best model to finance healthcare.

    4. I like the idea of self care and active care participation and management with the patient and the doctors. If physicians were adequate paid/reimbursed for providing email, telephone, and group consults it would be a far better use of their time, talent, and expertise. Patients do better when they are actively involved, but we’ve been trained to not be involved and to assume “Doctor knows best.”

    5. No gov’t involvement? Does he mean he doesn’t want licensure of professionals and hospitals? That’s done by government. It’s what currently grants MDs a monopoly on the practice of medicine. Is he prepared to let anyone practice medicine and bill for services? What about drug regulation? OR does he just mean in the financial arena? If there were no Medicare/Medicaid/TriCare most physicians would have half the income they currently have. Older patients would die sooner and have more disabilities. The elderly are at extremely high risk of not having adequate coverage and thus avoiding care. As the insurance agent says to me, why would I sell your home owners insurance if you house is burning…. Old patients are the equivalent of burning houses.

    In 1964 there were 40 million uninsured folks—most where over the age of 65 and lost their health insurance when they retired. Medicare allowed our grandparents and parents access to healthcare and a long life with significant dignity. It made the rapid expansion of health technology possible and it probably allowed your physician friend to graduate from Medical school. Not bad. Could it have done better? Yes. Less expensively? Probably. But it was the right thing to do for a generation who suffered the Great Depression and paid for World War II and who fought WWI and WWII.

    Does he want gov’t to stop funding higher education? That’s where doctors (lawyers, and consultants, too) come from. What about the public funding through nonprofit status of hospitals? Government in public health activities like water and sewer, hazardous waste disposal, poison control, restaurant inspection, etc. Gov’t is in a lot of places that touch healthcare.

    6. How does he suggest society prevent discrimination against patients by race, gender, age, nationality or the poor? I’m old enough to remember segregated hospitals. One of the main reasons Southern Senators and Congressmen voted against Medicare in 1965 was because it would force the integration of hospitals and other participating providers. In the 1950s and 1960s many physicians routinely avoided treating the poor Blacks except through charity or public clinics/hospitals. Is it really OK to discriminate against the sick and the ill because they don’t have money?

    Is that the type of society we really want? That is a high risk public health policy. The more equitable our healthcare the less likely we’ve been to have an epidemic. Think whooping cough, or mumps or measles, all have recently become local epidemics because of lack of access to care or the choice to avoid care. Your kids go to school don’t they? You interact with lots of folks at WalMart or the Kroger? Running and hiding isn’t a real option in today’s society.

    7. We totally need to reorganize medical education. And we could be much more creative in using teaching hospitals and academic medical centers in caring for low income patients. It is time for another Flexner Study. He is correct we don’t have enough primary care physicians. But educating physicians is expensive, they need lots of lab space, classrooms, and clinical supervision. All very expensive. We’ve not added an new medical school in the US since the late 1970s when we added the USC School of Medicine and Eastern Carolina School of Medicine. (Thank you Jesse Helms and Strom Thurmond.) Educating physicians for 7 years costs approximately $500,000 and tuition pays only about 1/3 of it. If society wants more MDs, we’re going to have to pay for it. That’s the reality of it.

    8. Drug costs are a function of how the US regulates intellectual property and the high costs come from how we grant patents and create monopolies. Most of the pharmaceutical profits and this is a truly global industry, come from sales of drugs in the US market. All other industrialized markets “regulate” drug supply and prices.

    9. Tort reform…oh were to start. An ego clash of two bright groups of professionals, physicians and lawyers. The best, academically rigorous studies don’t support the contention that malpractice and defensive medicine drives costs. Maybe only 1% to 2%, that’s all. Tort reform and ending fraud and abuse won’t pay to cover all uninsured folks.

    Physicians order lots of test and procedures because the technology is available and the culture of their colleagues encourages it, no one wants to be the last doctor on his block without the doing or having the newest “toy.” (I know this because my job is to get them the franchise to have these new toys and to be able to bill for these new toys. They love new toys. They are gadget happy. Who were the first to have Ipods and flat screens TVs in your neighborhood…the doctors. They can’t function in an OR without flat screen digital imaging systems. ) In the healthcare it really isn’t the BIG toys but the thousands and thousands of little moderately expensive toys that drives costs. This is the stuff that mushrooms everywhere you turn. Drs. are also drug happy…new drugs are better. This is just another new technology.

    All that being said, I’m all in favor of reforming malpractice but there has to be a system of assuring patient safety and quality of care. The adversarial system of resolving negative encounters with the healthcare system is toxic to all…patients, hospitals, and physicians. Physicians ought not be penalized when they make a judgment call and it hits the worst probability. Stuuf happens but we need to assure everyone that there are checks in the systems that protect all against sloppy, lazy, and stupid errors. So how do we get accountability, and corrective actions without arbitrarily punishing professionals? That I fear is a work in progress. ”

    I am all for tort reform. But don’t think that will control cost escalation.

  81. Country Doc says:

    Lee, your friend says much that is true. I only gave an outline, The Devil is in the details! To deal with each of his points can be done quite easily, but not on a blog. BTW, I was not advocating getting rid of government in medicine. I especiall believe in public health which has been quite successful. I just want them out of the insurance business, but I admitted that it was too late to return to pure Constitutional government, hence I gave points on how the government could be helpful without destroying our freedom.
    After some thought I have added two more points to round out the program:

    11. For catastrophic, devastating illness such as head injury, brain surgery, cardiac surgery, long term cancer and others as deemed necessary, there could be a program like the dialysis program now, that seems to work well. These conditions would be “Federalized” and the government would pick up most all the costs. They could have accredited centers of excellence that they could negotiate the price ahead of time like dialysis is done now. This would get the cost down and quality up. There would have to be centers all over the country. We have a trauma center network with different levels of complexity. Now anyone could go where ever they desired, but full payment would not be assured.
    12. The free ER window would have to be closed. If someone came in with a non-emergency or routine problem, then there would be a charge. If they had no insurance, then there would be trained social worker/financial councilors (funded by the government) who would sign them up and get them enrolled at a primary care clinic or Federal Health Clinic. If they came in twice uninsured for non-emergency care there would be a fine taken out of their welfare check or other government money they were receiving. ER’s would have sovereign immunity.
    I wish we could all sit down and have a discussion and that the powers that be would listen, but that will never be. I really believe our nation is under judgment. I also think that is true of TEC and Anglicanism, but I have no plan for saving them. Maybe like going to Costa Rico for doctors, going to the Presbyterian churches is the answer for them. Glad I don’t have to run that problem.

  82. Lee Parker says:

    Great comments Country Doc. We are under Judgement. I have seen grass root ground swells before and they start just like this. I’m praying about it. BTW I’m a Presbyterian now.

  83. Clueless says:

    #79: Self care: Agree. Except for the wimpy loser comment “but we’ve been trained to not be involved and to assume “Doctor knows best.”

    Yeh. Like you never had an independant thought with your your mommy either.

    2. No gov’t involvement? Does he mean he doesn’t want licensure of professionals and hospitals?

    I’m happy to give up the monopoly. Bring it on. No licensure, no regulation. Bring it on. I can compete with Aunt Millie. If not, I deserve to be put out of business.

    3. “Medicare allowed our grandparents and parents access to healthcare and a long life with significant dignity. It made the rapid expansion of health technology possible and it probably allowed your physician friend to graduate from Medical school. ”

    No. We took care of grandparents in the charity hospitals. They appreciated us, we appreciated them. Medical school cost less than 200 dollars. That was in the days that medicine was an apprenticeship.

    The regulation came in because it was easier to tax the doctors when the government paid them for Medicare, underwrote their increasing school expenses, and their increasing licensure fees. All that gets taxed. Medicine was initially a huge a provider of taxes to the federal government, then medicare spending suddenly made it a tax loss.

    “Does he want gov’t to stop funding higher education? That’s where doctors (lawyers, and consultants, too) come from. What about the public funding through nonprofit status of hospitals? ”

    Yes to all. Stop regulating medical schools. The “funding” is a joke. Like I said, 6 medical students in our biggining class are each paying 40 grand, and all the docs are volunteering their time, and the hospitals are volunteering space. But medical school costs 45 million because of regulation. Then they give us 15 million back in rebates and say that we need them to “subsize” medical education.

    6. ” How does he suggest society prevent discrimination against patients by race, gender, age, nationality or the poor? ”

    The same way society prevents discrimination in restaurants, bars and swimming pools. It seems to work.

    ” In the 1950s and 1960s many physicians routinely avoided treating the poor Blacks except through charity or public clinics/hospitals. Is it really OK to discriminate against the sick and the ill because they don’t have money? ”

    What is wrong with physicians working in charity hospitals? I think it is swell to give charity in the most cost effective setting. Why does your friend believe that he has a right to charity at cadillac prices? The person working for free gets to choose where and how much. If not, I would like your friend to come be my housekeeper for free, in the conveniance of my home, and at the times that are best for me, because I deserve it and he shouldn’t discriminate against me.

    ” Your kids go to school don’t they? You interact with lots of folks at WalMart or the Kroger? Running and hiding isn’t a real option in today’s society.”

    We didn’t have plagues in the days of the charity hospitals either. That is thanks to penicillin, and plumbing not medicare or medicaid.

    7. “We totally need to reorganize medical education. ”

    We can begin by getting the government out of it. See above regarding who drives up costs.

    8. Drug costs are a function of how the US regulates intellectual property and the high costs come from how we grant patents and create monopolies. Most of the pharmaceutical profits and this is a truly global industry, come from sales of drugs in the US market. All other industrialized markets “regulate” drug supply and prices.”

    All other countries sponge off our pharmaceutical sector. That is the only reason we have more than sulfar penicillin and bromides.

    9. Tort reform…oh were to start. An ego clash of two bright groups of professionals, physicians and lawyers. The best, academically rigorous studies don’t support the contention that malpractice and defensive medicine drives costs. Maybe only 1% to 2%, that’s all. Tort reform and ending fraud and abuse won’t pay to cover all uninsured folks.

    It is only 1-2% if you count payouts. If you count the amount of defensive medicine it is about 30%.

    Physicians order lots of test and procedures because the technology is available and the culture of their colleagues encourages it. No one wants to be the last doctor on his block without the doing or having the newest “toy.” ”

    Scarcely. We are expected by our patients to provide the “best” and sued if we fail to meet expectations. We therefore provide our patients with the best and charge them for it. If they want something different, then expectations will need to be lowered, and there can be no retaliation for failing to provide the MRI or mammogram that turned out to NOT be “unnecessary care” after all.

  84. Country Doc says:

    Cluless, you sayeth true sooth.

  85. Mitchell says:

    As a small businessman, who has been uncertain as to whether we need or can afford government health care or whether there are other options to contain the cost of health care, posts like the one submitted by clueless really help me focus on the options. I can now truly say that I would rather have any health care system in western Europe, including the system in the UK, or Canada than the draconian third world system proposed by clueless.

    With the exception of a few well funded organization, charity health care has always been crap. Poor people were cared for in charity wards and their care was significantly substandard, when compared to folks with money or insurance. Do people really believe hip and knee replacements will be available in charity wards? Do people really believe dialysis for old people will be available in charity wards? Do people really believe the latest drugs and procedures will be available in charity wards? How much money is a charity going to spend on a guy with a 30% survival rate, when they can treat ten other people who are likely to survive. If you are a charity will you give the heart to the 15 year old son of a uninsured secretary or the 70 year old multimillionaire willing to make a significant donation to your charity. In fact organ auctions could be a major fund raiser for charity hospitals. If we are going to deregulate medical care, why not? I understand a kidney goes for 20k on the black market. Imagine how much you could raise at a well publicized auction. Charities could organize organ donation fund raisers. Or you could pay for your son’s operation with a kidney or a lung.

    Doctors are no more charitable than the next guy. Most do what they do for money, and that is fine with me. But do not pretend we would have docs lining up to work in charity wards for crappy pay.

    Yes you can reduce the cost of health care under a system like that proposed by clueless. In fact I have personally visited several places where that system is in operation right now. Medical care is largely deregulated, there are plenty of nontraditional practitioners, plenty of alternative medications, drugs are largely available without prescription, people make most of their own Medical decisions based on their life experiences, Doctors are consulted only when a family determines the condition is very serious, there are charities who help those who cannot afford care, and tort law is virtually nonexistent. Examples are Calcutta, Haiti, Bangladesh, and the jungles of Guatemala and Honduras.

  86. Clueless says:

    ” I can now truly say that I would rather have any health care system in western Europe, including the system in the UK, or Canada than the draconian third world system proposed by clueless. ”

    Unfortunately, none of the European states, or Canada can afford their health care systems either. They too are desperately looking for savings because their systems are also unaffordable, and becoming more so, as their population ages. Their systems are just not collapsing as fast as our system.

    “Medical care is largely deregulated, there are plenty of nontraditional practitioners, plenty of alternative medications, drugs are largely available without prescription, people make most of their own Medical decisions based on their life experiences, Doctors are consulted only when a family determines the condition is very serious, there are charities who help those who cannot afford care, and tort law is virtually nonexistent. Examples are Calcutta, Haiti, Bangladesh, and the jungles of Guatemala and Honduras. ”

    I have (limited) experience of a similar system in Sri Lanka. If you have an educated population it is not too bad. The problem is folks who simply have no clue as to how to manage medications. Medication management, and self prescription, however is a subject that is fairly easy to teach.

    I went to medical school after the death of the charity hospitals, but by all accounts, charity care worked well in the United States. It was simply felt to be “undignified” for folks to receive “charity” care. Thus, Medicaid was concocted, which pays less than costs and therefore is essentially charity care from the point of view of the physicians, and from the point of view of the private insurance holders who subsidize it. However, because Medicaid is dispensed in a private setting, it has the feel of not being charity, thus permitting medicaid recipients to be excused the “burden” of gratitude.

    Whether an ungrateful, litigenous patient receiving charity is more “dignified” than a grateful, cooperative patient is not clear to me.

  87. Country Doc says:

    Mitchell, I resent your charge that charity hospitals and clinics are uncaring, inferior, and cruel. I have worked in such places and found the care on a par with the private sector. The decor and ammenities were different, but no ethical doctor will tolerate substandard care. Pedal your ideas at the Shriners Hospitals or St. Jude’s in Memphis. They refuse to charge and are the best. Those third world countries have the problems you name not becaused of poor medical care, but due to the heathen culture, ignorance, non-compliance, and war and turmoil. If you like the Obama Care type schemes, then why aren’t you insisting that we not limit it to the US, but provide it for all the world? The four billion in endowments of the Episcopal Church would go a long way to help. I have worked in such places. Lack of money, not medical care is the root of the problem. Corrupt government and communism/socialism will assure a lack of money and hence lack or food, medicine, safety, evangelism. etc. People will die tonight for the lack of $25 dollars. Sheeesh!

  88. Clueless says:

    “Corrupt government and communism/socialism will assure a lack of money and hence lack or food, medicine, safety, evangelism. etc. People will die tonight for the lack of $25 dollars. Sheeesh!”

    Preach it Bro! AMEN!

  89. Mitchell says:

    Country Doc:
    I am leaving work so I cannot address all of your charges. But first I never said anyone was uncaring or cruel. Obviously people who chose to work for reduced pay in the charity ward of a hospital is not cruel or uncaring. Second, if you read my post you will see that I was not even addressing modern well funded charitable hospitals. Of course the ones you name are excellent facilities. I was addressing the charity wards that existed to care for the poor from the mid 1800s though the 1960s. The ones someone in this thread said were put out of business by Medicare, and should be brought back to replace Medicare and Medicaid. You will also see that in addition I specifically excepted from my post certain well funded organizations. I would include in that group the ones you list and many of the Catholic hospitals. The institutions you describe were clearly the exceptions to the norm.
    But if you are contending poor people in the 30s, 40s, and 50s received the same care in the charity wards of rural hospitals and underfunded urban hospitals as people with money, I simply do not believe you are correct. If a retired cotton mill worker living in rural Mississippi in the 1950s contract a serious illness, I assure you he did not receive the same care as a wealthy Wall Street executive with the same disease. In fact he would have been lucky if I even saw a competent doctor before he died, much less received cutting edge treatment. Medicare, or socialized medicine as you call it, went a long way toward changing that.

    If you will also note, I said I would rather have a European health care system than the health care system Clueless described. Which is the system we had in this country from the time of the Great Depression through most of the 1950s. And yes I believe MOST poor people during that time period received inferior health care. And yes I believe ethical doctors allowed substandard care to exist for some because it was better than no care, and they had no other options.
    Finally do I believe if we went back to a charity based system and eliminated Medicare and Medicaid people with money would get better care than those without money? Yes I do.

    Ultimately I guess I believe rationing health care on the basis of wealth is immoral and the product of a heathen culture as you would call it. But I accept that is the way of the world. I would like to reduce that factor as much as possible in this country, but I cannot extend that goal to the rest of the world, because I haven’t the resources.

  90. Clueless says:

    ” And yes I believe MOST poor people during that time period received inferior health care. And yes I believe ethical doctors allowed substandard care to exist for some because it was better than no care, and they had no other options.”

    For emergencies, (appendectomies) and broken bones) charity care was not inferior care. For routine care it was long waits in shabby facilities, yes.

    However the problem is that because charity care has been replaced by Medicaid/Medicare as physicians drop both, all care will vanish, unless the algorith is changed. Already, medicaid patients in my part of the woods are unable to get any kind of care beyond the hospital emergency room without driving 300 miles. Physicians simply can’t afford to pay 100 dollars in office costs and malpractic insurance to see somebody who brings in 10 dollars. I was the last person seeing kids in my area before I bailed out. As it is, physicians are dropping medicare (which also doesn’t (quite) cover costs.

    So without bringing back charity care, as the health care crisis continues we will go through no care. The proposals in Congress simply suggest increasing government care (which already does not pay overhead) to even more people. It can’t work unless there is rationing by access (no care outside of the ER or else drive 300 miles, which is what medicaid folks do now) or rationing by age (no care over 70) or explicit rationing (no care for dialysis, no care for transplants etc). The proposal in Congress looks good because it promises insurance coverage, but it doesn’t promise that there will be somebody there to see the patient with medicaid/medicare/obamacare. Having a card that says “Medicaid” or “Obamacare” doesn’t mean that you can actually find a surgeon who will fix your shoulder.

  91. Clueless says:

    People seem to think that universal healthcare can be achived with a simple fix: Tax the “rich” more. Make doctors see everybody for free. Stop “fraud and abuse”. They think it is possible to maintain the current system, which only works by increasing the cost of subsidized (medicaid/medicare/government) medicine that are paid for by those who pay for private insurance.

    America’s middle class and business can’t afford to have higher private insurance premiums, and those on government subsidy don’t want to give up what they get.

    In the absence of useful strategies to ensure that people will still be able to get their BP medications filled, this will lead (slowly but inexorably) to no care. Physicians will first drop medicaid, then drop medicare, then (if forced by condition of licensure to see people) retire. It is already happening. Hospitals will close. People will drive further for care, because they can’t get it in town. They will wonder (vaguely) why and assume that it somehow has something to do with greed.

    In the past 6 months 50 of the fewer than 300 physicians in my area have left private medicine. None of them have been replaced in the private sector though the VA has gotten 30 of them. Those 30 were the cream of the crop, not the dregs.

    The current system is unsustainable, and the proposals in congress will make it worse. That is the problem.

  92. Mitchell says:

    Clueless, I think we want the same thing. Quality health care for as many people as possible. I just think we differ on how to determine who gets the care. I had hoped there was something between pure capitalism and socialized medicine, but the more I hear from both sides, the more I fear there may be no sustainable middle ground. Further, I must admit our nations attempt to create a middle ground over the last 40 years is failing. That said I cannot see myself supporting a health care system, where the size of ones bank account is the primary factor in determining who lives and who dies.

    As far as making doctors see people for free, I do not know anyone who wants that. As I said earlier that is charity care, and in the end that is generally poor care. Just look at the legal profession. The state makes lawyers represent criminals for free if they can’t afford a lawyer, and no one wants a court appointed attorney. You might end up with a tax lawyer fighting to keep you out of prison.

    As for taxing the rich, I think it was a mistake for Obama to promise he would not raise taxes on anyone making less than 250k. I think his opponents made that promise necessary, and continue to make it necessary. I think any necessary tax increase should be across the board, but that would not have appeased those in our society who have vowed for the last 25 years to allow no more tax increases, no matter what happens, forever.

    From the problems you and other on this thread have identified, it does seem many of our problems could be alleviated if we dramatically increased the supply of doctors, by increasing the supply of medical schools and making it easier for doctors trained in other countries to come here to practice. While American doctors are leaving the practice because of low pay, maybe Indian doctors would appreciate the raise. Would that mean lower quality. I don’t know. It may just mean fewer hours. Maybe some docs would not object to lower pay if they worked 40 hour weeks.

    Also Rather than spending billions of dollars on cash for clunkers, and stimulus spending, the government could have committed to building and staffing 20 new Medical Schools over the next ten years. This would increase competition among schools for students resulting in lower tuition, and increased competition for jobs and patients resulting in lower fees; but as an offset fewer work hours. If there were not enough US students to fill these schools open them up to foreign students, with offers to let them practice here when they graduate. That way we can also control the quality of education. I am certain their are plenty of very intelligent people in India, China, Africa, and South America who would love to be trained and work in the US for significantly more than they could ever earn in their country. Plus this is supply sider solution to the cost problem. Conservatives should appreciate that.

  93. Clueless says:

    “Indian doctors would appreciate the raise. Would that mean lower quality. I don’t know. It may just mean fewer hours. Maybe some docs would not object to lower pay if they worked 40 hour weeks”

    My Indian collegues are going home. Their quality of life is better there. Although their pay is less, it is still sufficient to permit them to send their kids to private schools, live in a mansion and to engage six servants. England’s Indian physicians are also going home. The reason that Indian physicians came to the West (not counting those who married, had kids in the US and made it their home) is not because their quality of life is better in the US. It never was. They came because they could work at a higher level, and feel more fulfilled, and could save money which they would then send home to India where they planned to eventually retire, where they could live like kings in retirement. My brother’s father in law (an oncologist originally trained in India) has closed up shop and is going home and he does have 3 kids and a number of grandkids who are US citizens, and plan to stay. Short of a civil war in India I do not see that calculus changing.

    We may be able to get physicians from Africa for cheaper. I ask, however, if it is fair for Africa’s poor to pay taxes to pay for their free medical schools and then have the US, who is too cheap to pay for physician medical education (taxing it heavily instead -see above) to take the finished product?

    All other countries pay for their physicians medical education. The US is an outlier.

    I still think we’d do better giving prescription powers to everybody for most, nonaddictive agents.

    As for getting lower pay and working 40 hour weeks, how will that work? I would have loved to have a 40 hour week and break 50,000 in private practice. When I was in private practice, if I worked 40 hours, I owed my practice about 20,000. If I worked 60 hours a week, I broke even on expenses but didn’t get a salary. By working 80 hours/week I managed to earn about 80,000, which after paying for night coverage for children did not go very far.

    The US cannot afford the about of subsidized government care she already offers (medicaid and medicare). Adding another layer of subsidized folk to the backs of the physicians and private insurance holders will result in implosion. It is already happening.

    I would like everybody who feels that they would benefit from Obama care to do the following experiment:

    Call up 10 internists or family physicians in your local area and say “I am looking for a new family physician and I have medicaid. Does Dr. X accept new medicaid patients?”

    If you find that it is easy to find a practioner to take you, then you should have no trouble with Obamacare. If not, then I would seriously rethink your position about whether you wish to have governmental care or whether you would rather have SOME care, even if it isn’t subsidized.

  94. Jeremy Bonner says:

    [i]I still think we’d do better giving prescription powers to everybody for most, nonaddictive agents.[/i]

    Clueless,

    My wife is a hospital pharmacist. Much of her job is advising surgeons as to potential consequences of drug interactions (they don’t have time to follow all the literature, which is voluminous). With the explosion of drug options in recent years, one has increasingly to deal not only with the individual effects of a drug but the multiple effects of many drugs taken together.

    In the context you described of some care being better than none, I suppose self-prescription is a possible solution, but I wonder if it’s as easy as someone (possibly you) implied earlier in the thread, with reference to working out medical solutions by trial and error.

  95. Clueless says:

    We are expected to manage our own taxes. The IRS code is no more complex than is pharmaceuticals.

    It is true that there are thousands of drugs out there. However most patient don’t have thousands of illnesses. They have 2-3 (the older they get, the more they have, and most folks over 80 have 8-9 illnesses).

    One begins at 40 by developing hypertension. One then reads the various algoriths on hypertension, which requires a choice of 4-5 agents. One studies, does one’s homework, monitors ones BP and reads up on side effects.

    One also gets one’s routine labs (algoriths for what is appropriate for age are readily available).

    At 50 one develops elevated cholesterol. Again one studies about cholesterol medications. One checks (on line drug checkers are already available) to see if your proposed cholesterol medication interacts with your previously chosen antihypertensive agent. One monitors for side effects.

    Yes, it does require work. However it would be a lot better than a health care system where only physicians write prescriptions, and there are no physicians. The latter system is what medicaid patients have, and is the system we are all hurtling toward.

  96. Clueless says:

    Mitchell wrote; “the government could have committed to building and staffing 20 new Medical Schools over the next ten years. This would increase competition among schools for students resulting in lower tuition”

    Are you reading anything I write? The ONLY reason medical school is so expensive is the government. Physicians, and hospitals give their space and teach for free. We have always done so, since the days of the physician apprentices. If the government wants more medical schools, all it needs to do is to stop adding 45 million dollars in taxes/regulation on them. Then medical school would cost less than community college, and yeah, there’d be a lot more folks who would be interested in applying.

    However, those taxes are part of what pays for medicaid and medicare – Which are still bankrupting the government despite the taxes on medical schools (not to mention on hospitals and physicians).

    Eventually you run out of other peoples’ money.

  97. Clueless says:

    http://www.npr.org/templates/story/story.php?storyId=16774871

    Here is an article about why Indian physicians came, and why they are now planning to return home to India. I think it is fairly written, and I anticipate more Indian physicians returning home.

    Me, I’m from Sri Lanka, but I was born here and I went to school here. This is my home. I don’t see myself going back even though I probably could improve my lifestyle.